2 DIFFICULT QUESTIONS

This topic has expert replies
Senior | Next Rank: 100 Posts
Posts: 43
Joined: Thu Sep 11, 2008 10:05 pm
Location: India

2 DIFFICULT QUESTIONS

by poonam197 » Fri May 01, 2009 3:41 am
Brownlea’s post office must be replaced with a larger one. The present one cannot be expanded. Land near the present location in the center of town is more expensive than land on the outskirts of town. Since the cost of acquiring a site is a significant part of the total construction cost, the post office clearly could be built more cheaply on the outskirts of town.

Which one of the following, if true, most seriously undermines the argument’s stated conclusion?

(A) The new post office will have to be built in accordance with a demanding new citywide building code.
(B) If the new post office is built on the outskirts of town, it will require a parking lot, but if sited near the present post office it will not.
(C) If the new post office is built on the outskirts of town, current city bus routes will have to be expanded to provide access.
(D) If the new post office is built on the outskirts of town, residents will make decreased use of post office boxes, with the result that mail carriers will have to deliver more mail to homes.
(E) If the new post office is built near the center of town, disruptions to city traffic would have to be minimized by taking such steps as doing some construction work in stages at night and on weekends.

Answer: B

In the past, the railroads in Ostronia were run as regional monopolies and operated with little regard for what customers wanted. In recent years, with improvements to the Ostronian national highway network, the railroad companies have faced heavy competition from long-distance trucking companies. But because of government subsidies that have permitted Ostronian railroad companies to operate even while incurring substantial losses, the companies continue to disregard customers’ needs and desires.

If the statements above are true, which one of the following must also be true on Fthe basis of them?
(A) If the government of Ostronia ceases to subsidize railroad companies, few of those companies will continue to operate.
(B) Few companies in Ostronia that have received subsidies from the government have taken the needs and desires of their customers into account.
(C) Without government subsidies, railroad companies in Ostronia would have to increase the prices they charge their customers.
(D) The transportation system in Ostronia is no more efficient today than it was in the past.
(E) In recent years, some companies in Ostronia that have had little regard for the desires of their customers have nonetheless survived.

Answer: E

Legendary Member
Posts: 1035
Joined: Wed Aug 27, 2008 10:56 pm
Thanked: 104 times
Followed by:1 members

by scoobydooby » Fri May 01, 2009 4:03 am
1) to show that post office in the outskirts will not be cheap

A. not given if it woud impact the cost at all

B. B is better than D as parking lot will require land, significant part of the construction, so it will most definitely impact cost

C. doesnt weaken. post office will not bear the cost of expansion of bus services.

D. close. mail carrier cost will increase, we do not know if it is a significant part of the total construction cost, so loses out to B

E. doesnt weaken as doesnt say if construction in stages would impact cost adversely.

hence, B


2)
A. based on the info given, who knows what would they do if the govt. stops the subsidies.

B. no info in passage to support this.

C. based on the info given, who knows what would they do if the govt. stops the subsidies.

D. passage doesnt talk about efficiency of the transport system in general

E. true, railroad companies ingnored customer needs in the past and continue to do so now. (1st and last lines)

Senior | Next Rank: 100 Posts
Posts: 43
Joined: Thu Sep 11, 2008 10:05 pm
Location: India

by poonam197 » Thu May 07, 2009 7:45 am
Could you help me with below 2 questions too. Thanks !

Members of the Amazonian Akabe people commonly take an early-morning drink of a tea made from the leaves of a forest plant. Although they greatly enjoy this drink, at dawn they drink it only in small amounts. Anthropologists hypothesize that since this tea is extraordinarily high in caffeine, the explanation for the Akabe’s not drinking more of it at dawn is that high caffeine intake would destroy the surefootedness that their daily tasks require.
Which one of the following, if true, most seriously calls the anthropologists’ explanation into question?

(A) The drink is full of nutrients otherwise absent from the Akabe diet.
(B) The Akabe also drink the tea in the evening, after their day’s work is done.
(C) The leaves used for the tea contain a soluble narcotic.
(D) Akabe children are introduced to the tea in only a very weak form.
(E) When celebrating, the Akabe drink the tea in large quantities.

Answer: Option C


Ann will either take a leave of absence from Technocomp and return in a year or else she will quit her job there; but she would not do either one unless she were offered a one-year teaching fellowship at a prestigious university. Technocomp will allow her to take a leave of absence if it does not find out that she has been offered the fellowship, but not otherwise. Therefore, Ann will quit her job at Technocomp only if Technocomp finds out she has been offered the fellowship.
Which one of the following, if assumed, allows the conclusion above to be properly drawn?
(A) Technocomp will find out about Ann being offered the fellowship only if someone informs on her.
(B) The reason Ann wants the fellowship is so she can quit her job at Technocomp.
(C) Technocomp does not allow any of its employees to take a leave of absence in order to work for one of its competitors.
(D) Ann will take a leave of absence if Technocomp allows her to take a leave of absence.
(E) Ann would be offered the fellowship only if she quit her job at Technocomp.

Answer: Option D

User avatar
Senior | Next Rank: 100 Posts
Posts: 51
Joined: Wed Apr 29, 2009 9:00 am
Thanked: 6 times

by anksgupta » Thu May 07, 2009 11:52 am
Members of the Amazonian Akabe people commonly take an early-morning drink of a tea made from the leaves of a forest plant. Although they greatly enjoy this drink, at dawn they drink it only in small amounts. Anthropologists hypothesize that since this tea is extraordinarily high in caffeine, the explanation for the Akabe’s not drinking more of it at dawn is that high caffeine intake would destroy the surefootedness that their daily tasks require.
Which one of the following, if true, most seriously calls the anthropologists’ explanation into question?

(A) The drink is full of nutrients otherwise absent from the Akabe diet.
(B) The Akabe also drink the tea in the evening, after their day’s work is done.
(C) The leaves used for the tea contain a soluble narcotic.
(D) Akabe children are introduced to the tea in only a very weak form.
(E) When celebrating, the Akabe drink the tea in large quantities.
IMO C should be the correct ans.

A. If at all drink is full of nutrients, there is no reason to take it in less quantity. So A is not correct
B.Drinking tea in evening is an independent event ans is not related to quantity of tea consumption. So B is incorrect
C. Gives a valid reason that narcotics rather than caffeine is teh reason, why Akabe drinks less tea at dawn. Correct ans
D. Not related to the question
E. Although seemingly correct, this doesn't undermines the author assumption.

Legendary Member
Posts: 876
Joined: Thu Apr 10, 2008 8:14 am
Thanked: 13 times

by ketkoag » Fri May 08, 2009 4:45 am
poonam197 wrote:Could you help me with below 2 questions too. Thanks !

Members of the Amazonian Akabe people commonly take an early-morning drink of a tea made from the leaves of a forest plant. Although they greatly enjoy this drink, at dawn they drink it only in small amounts. Anthropologists hypothesize that since this tea is extraordinarily high in caffeine, the explanation for the Akabe’s not drinking more of it at dawn is that high caffeine intake would destroy the surefootedness that their daily tasks require.
Which one of the following, if true, most seriously calls the anthropologists’ explanation into question?

(A) The drink is full of nutrients otherwise absent from the Akabe diet.
(B) The Akabe also drink the tea in the evening, after their day’s work is done.
(C) The leaves used for the tea contain a soluble narcotic.
(D) Akabe children are introduced to the tea in only a very weak form.
(E) When celebrating, the Akabe drink the tea in large quantities.

Answer: Option C


Ann will either take a leave of absence from Technocomp and return in a year or else she will quit her job there; but she would not do either one unless she were offered a one-year teaching fellowship at a prestigious university. Technocomp will allow her to take a leave of absence if it does not find out that she has been offered the fellowship, but not otherwise. Therefore, Ann will quit her job at Technocomp only if Technocomp finds out she has been offered the fellowship.
Which one of the following, if assumed, allows the conclusion above to be properly drawn?
(A) Technocomp will find out about Ann being offered the fellowship only if someone informs on her.
(B) The reason Ann wants the fellowship is so she can quit her job at Technocomp.
(C) Technocomp does not allow any of its employees to take a leave of absence in order to work for one of its competitors.
(D) Ann will take a leave of absence if Technocomp allows her to take a leave of absence.
(E) Ann would be offered the fellowship only if she quit her job at Technocomp.

Answer: Option D
could you please let us know the source of these questions, as i am taking the test very soon, the source might help me if its the gmat source not the lsat source..
thanks.

Senior | Next Rank: 100 Posts
Posts: 43
Joined: Thu Sep 11, 2008 10:05 pm
Location: India

by poonam197 » Fri May 08, 2009 6:05 am
The questions are from 1000CR file and they fall under the LSAT section.

Master | Next Rank: 500 Posts
Posts: 232
Joined: Fri Jul 04, 2008 4:14 pm
Thanked: 14 times
Followed by:1 members
GMAT Score:760

by rs2010 » Fri May 08, 2009 7:28 am
Ann will either take a leave of absence from Technocomp and return in a year or else she will quit her job there; but she would not do either one unless she were offered a one-year teaching fellowship at a prestigious university. Technocomp will allow her to take a leave of absence if it does not find out that she has been offered the fellowship, but not otherwise. Therefore, Ann will quit her job at Technocomp only if Technocomp finds out she has been offered the fellowship.
Which one of the following, if assumed, allows the conclusion above to be properly drawn?
(A) Technocomp will find out about Ann being offered the fellowship only if someone informs on her.
(B) The reason Ann wants the fellowship is so she can quit her job at Technocomp.
(C) Technocomp does not allow any of its employees to take a leave of absence in order to work for one of its competitors.
(D) Ann will take a leave of absence if Technocomp allows her to take a leave of absence.
(E) Ann would be offered the fellowship only if she quit her job at Technocomp.
B and E are about quiting job so let keep them aside.

A tells about the how would Technocorp find out---
C tells reasoning why?
D about the scenario when we can apply this theory. If Ann is on leave then only later picture will have a part to play. This should be our assumption.

Master | Next Rank: 500 Posts
Posts: 154
Joined: Mon Apr 13, 2009 10:57 pm
Thanked: 3 times

by thetrystero » Sun May 10, 2009 9:02 pm
In the past, the railroads in Ostronia were run as regional monopolies and operated with little regard for what customers wanted. In recent years, with improvements to the Ostronian national highway network, the railroad companies have faced heavy competition from long-distance trucking companies. But because of government subsidies that have permitted Ostronian railroad companies to operate even while incurring substantial losses, the companies continue to disregard customers’ needs and desires.

If the statements above are true, which one of the following must also be true on the basis of them?
(A) If the government of Ostronia ceases to subsidize railroad companies, few of those companies will continue to operate.
(B) Few companies in Ostronia that have received subsidies from the government have taken the needs and desires of their customers into account.
(C) Without government subsidies, railroad companies in Ostronia would have to increase the prices they charge their customers.
(D) The transportation system in Ostronia is no more efficient today than it was in the past.
(E) In recent years, some companies in Ostronia that have had little regard for the desires of their customers have nonetheless survived.

Situation:
1. Rail faces heavy competition from trucks.
2. This leads to heavy losses for rail.
3. Gov bails rail out.
4. Allowing rail to continue bad customer service while staying afloat.

A. keep
B. irrelevant
C. irrelevant
D. cannot generalize to all forms of transportation. we are not told anything about planes, ships etc.
E. keep

E basically says exactly what is stated in the situation, whereas for A, we don't know that they'll go under for sure. perhaps they'll buck up and improve their customer service, thereby improving business.

My answer: E

Master | Next Rank: 500 Posts
Posts: 197
Joined: Tue Apr 22, 2014 4:03 am

by nicolette » Sun May 15, 2016 1:03 pm
I believe the answer should be E

Master | Next Rank: 500 Posts
Posts: 100
Joined: Tue Aug 05, 2014 11:14 am

by Brad.C » Sun May 15, 2016 1:39 pm
I think for the first one answer is B for the second E